Integrating with ln in denominator.

  • Thread starter Thread starter Poop-Loops
  • Start date Start date
  • Tags Tags
    Integrating Ln
Click For Summary
SUMMARY

The discussion focuses on the integration of the function \(\int \frac{1}{n(1 + \ln{n})^{2/3}} dn\). The user proposes using the substitution \(u = \ln(n)\), which leads to \(du = \frac{1}{n} dn\). Another participant confirms this approach and suggests a further substitution \(1 + u = v^3\), simplifying the integral to \(3 \int dv\). The conversation emphasizes the importance of verifying the solution by differentiating the result to ensure it matches the original integrand.

PREREQUISITES
  • Understanding of integral calculus, specifically integration techniques.
  • Familiarity with substitution methods in integration.
  • Knowledge of logarithmic functions and their properties.
  • Ability to differentiate functions to verify integration results.
NEXT STEPS
  • Practice integration techniques involving logarithmic functions.
  • Explore advanced substitution methods in calculus.
  • Learn about convergence tests for improper integrals.
  • Review differentiation techniques to validate integration results.
USEFUL FOR

Students studying calculus, particularly those focusing on integration techniques, and anyone interested in understanding convergence of integrals involving logarithmic functions.

Poop-Loops
Messages
731
Reaction score
1
Ok, so it's been a while since I've had to integrate anything, much less something like this.

\int \frac{1}{n(1 + \ln{n})^{2/3}} dn

I'm thinking u substition for ln(n) and then du becomes 1/n? But, since the ln(n) is in the denominator of a fraction raised to a power, wouldn't that mess with du? Or am I on the right track? I checked my calculus book and surfed the web, and couldn't find an integral for ln, so I can't see any other way...

If it helps, this is for a series check by integration to see if it's convergant or divergant, so it goes from nothing to infinity.
 
Physics news on Phys.org
Substitute u = ln(n), then substitute 1+u = v^3, and you simply have the integral of 3 dv
 
Last edited:
You've got the right idea. Remember, a substitution is just that, substituting in another number or variable, so why would you be worried? :smile:
 
Because these things take a LONG time, and then I don't even know if I have the right answer unless I get stuck (then I know it's wrong).

Thanks. I'll play around with that tomorrow. I've been doing my homework for like 3 hours now, so I think I'll go to bed. :)
 
Also, remember that you can always check your answer by taking the derivative of your answer and seeing if it turns out to be the integrand

EDIT: Just by looking at it, it seems that your answer will be very straighforward and take very little steps to arrive at.

EDIT2: koroljov: you shouldn't simply provide the answer in such a situation.
 
Last edited:
Question: A clock's minute hand has length 4 and its hour hand has length 3. What is the distance between the tips at the moment when it is increasing most rapidly?(Putnam Exam Question) Answer: Making assumption that both the hands moves at constant angular velocities, the answer is ## \sqrt{7} .## But don't you think this assumption is somewhat doubtful and wrong?

Similar threads

  • · Replies 3 ·
Replies
3
Views
1K
  • · Replies 3 ·
Replies
3
Views
7K
Replies
8
Views
2K
  • · Replies 4 ·
Replies
4
Views
1K
  • · Replies 22 ·
Replies
22
Views
4K
  • · Replies 13 ·
Replies
13
Views
2K
  • · Replies 4 ·
Replies
4
Views
2K
Replies
11
Views
2K
Replies
1
Views
1K
  • · Replies 3 ·
Replies
3
Views
1K